Change of variables in a simple integral

Click For Summary
The discussion revolves around the change of variables in a simple integral involving the relationship between variables x, v, and their derivatives. A key point of contention is whether the integral should include a power of 1/2 or 3/2, with references made to a specific equation in Reif's textbook. The participants highlight the dependency of x and g on v^2 and the implications for partial derivatives. There is also a suggestion to verify the results through dimensional analysis, emphasizing the complexity of understanding the solutions alongside the material. The conversation reflects the challenges of mastering integral transformations in physics.
MathematicalPhysicist
Science Advisor
Gold Member
Messages
4,662
Reaction score
372
Homework Statement
I have the integral ##\int_0^\infty dv v^2 x \frac{\partial g}{\partial x}##.
Using ##x=\beta(\frac{1}{2}mv^2-\mu)## we get:
##\int_{-\infty}^\infty dx x (x+\beta \mu)^{3/2}\frac{\partial g}{\partial x}##.

The last integral upto some constants which don't get integrated.
Relevant Equations
they appear already in the HW statement.
So we have ##x=\beta(1/2 mv^2-\mu)##, i.e ##\sqrt{2(x/\beta+\mu)/m}=v##.
##dv= \sqrt{2/m}dx/\sqrt{2(x/\beta+\mu)/m}##.

So should I get in the second integral ##(x+\beta \mu)^{1/2}##, since we have: $$v^2 dv = (2(x/\beta+\mu)/m)\sqrt{2/m} dx/\sqrt{2(x/\beta+\mu)/m}$$

So shouldn't it be a power of 1/2 and not 3/2?

Thanks in advance!
 
Physics news on Phys.org
You show no explanation about ##\frac{\partial g}{\partial x}##.
 
anuttarasammyak said:
You show no explanation about ##\frac{\partial g}{\partial x}##.
Well, it's written that ##x## and ##g## both depend on ##v^2## only.
How does that fact change any thing here?
 
Ah, besides the fact that ##\partial g/\partial x = \partial g /\partial v \partial v / \partial x##, but I am not sure he changes it here that way...
 
I believe it should be 1/2 and not 3/2 because in the SM of Reif on equation (8) of page 108 he changes the LHS to 1/2 instead of 3/2.

Reading the SM alongside the problems and trying to understand the solutions can be quite time consuming, but hey that's what I chose to do. :-)
 
How about checking it by looking at dimension ? Apart from ##\frac{\partial g}{\partial x}## integral has dimension of ##L^4T^{-3}## if it is correct to take v and x velocity and coordinate.
 
Question: A clock's minute hand has length 4 and its hour hand has length 3. What is the distance between the tips at the moment when it is increasing most rapidly?(Putnam Exam Question) Answer: Making assumption that both the hands moves at constant angular velocities, the answer is ## \sqrt{7} .## But don't you think this assumption is somewhat doubtful and wrong?

Similar threads

  • · Replies 10 ·
Replies
10
Views
2K
Replies
4
Views
2K
  • · Replies 10 ·
Replies
10
Views
2K
  • · Replies 10 ·
Replies
10
Views
2K
  • · Replies 105 ·
4
Replies
105
Views
6K
  • · Replies 5 ·
Replies
5
Views
2K
Replies
4
Views
2K
  • · Replies 1 ·
Replies
1
Views
1K
Replies
3
Views
1K
  • · Replies 3 ·
Replies
3
Views
890